Print Page | Close Window

amp testing methods

Printed From: the12volt.com
Forum Name: Car Audio
Forum Discription: Car Stereos, Amplifiers, Crossovers, Processors, Speakers, Subwoofers, etc.
URL: https://www.the12volt.com/installbay/forum_posts.asp?tid=126051
Printed Date: April 28, 2024 at 1:44 PM


Topic: amp testing methods

Posted By: ianarian
Subject: amp testing methods
Date Posted: February 07, 2011 at 12:43 AM

Can someone please take a look at the figures and methods used on these bench tests and convey their opinion as is to if they feel that its valid or not?

https://www.soundsolutionsaudio.com/forum/topic/30180-uniform-amp-clamp-test/

-------------
This is what I do for FUN!



Replies:

Posted By: oldspark
Date Posted: February 07, 2011 at 4:01 AM
Assuming sinewave (else true-RMS clamps) and that only one polarity is feed thru the current clamp, it shows the volt/current output for a whatever signal input at whatever gain at whatever dc voltage.


It still amazes me that amps with their SMPS are so dependent on input voltage for max output power!




Posted By: i am an idiot
Date Posted: February 07, 2011 at 4:44 AM

I would be curious to find out how they "Calculated" the impedance of the speakers.  Impedance fluctuates drastically depending on frequency and box tuning.  I just went back and looked at the numbers.  They only read voltage and used the speakers impedance to get the rest of the numbers.  A 4 ohm speaker at resonance will read much higher than 4 ohms.  Hence much less current than they came up with. 

I am wondering why they calculated the current when they have a clampmeter connected to the wire.





Posted By: oldspark
Date Posted: February 07, 2011 at 3:45 PM
Not that I checked, but I assumed he used V/I - ie, rms impedance.

And maybe I should check it all again - I thought they were MEASURED values to provide 50Hz power & impedance.
(As to why and why not (say) 2 test frequencies & if 50Hz was not resonant etc, and not test distortion, closed/open environment....)





Posted By: ianarian
Date Posted: February 07, 2011 at 7:10 PM
Its the validity of the resulting wattage that concerns me...   I'm failing to see the mathematical proof of the values they are stating.

-------------
This is what I do for FUN!





Print Page | Close Window